You are on page 1of 33

www.iasprayojan.

com IAS PRAYOJAN NAGPUR


DETAILED SOLUTION
PRELIMS TEST: 17 (2020) TEST CODE: A-20117
SUBJECT: HISTORY – VI + CURRENT AFFAIRS (As scheduled)
----------------------------------------------------------------------------------------------------------------------------------------------
Q 1. (D)
 The Tangkhul-Naga tribe came to Manipur, Nagaland, Assam and Arunachal Pradesh from China through
Myanmar, entering their present habitats in successive waves of immigration from 9th to 11th century BC.
 Most of them live in Manipur's Ukhrul district in North East India.
 Luira phanit is a seed sowing festival celebrated by the Tangkhul Naga tribes.

Q 2. (B)
 Cholesterol is a waxy, fat-like substance produced in the liver.
 High density Lipoprotein (HDL) cholesterol and Low density Lipoprotein (LDL) cholesterol are the two main
types of cholesterol.
 LDL cholesterol is considered the ―bad‖ cholesterol, because it contributes to fatty build-ups in arteries
(atherosclerosis).
 This condition narrows the arteries and increases the risk for heart attack, stroke and peripheral artery
disease, or PAD.
 HDL cholesterol can be thought of as the ―good‖ cholesterol. (So, in the case of HDL cholesterol, higher
levels are actually better).
 Experts believe that HDL acts as a scavenger, carrying LDL (bad) cholesterol away from the arteries and
back to the liver, where the LDL is broken down and passed from the body.

Q 3. (D)
 Union Government has formed a Task Force for identifying areas for use of space technology in improving
border management.
 Details of the areas identified for use of space technology by the task force are as under:-
1. Island development and security
2. Border Surveillance
3. Communication and Navigation
4. GIS and Operations Planning System
5. Border Infrastructure Monitoring

Q 4. (D)
 The foundation stone for Sela Tunnel Project was laid recently.
 Sela Pass also called as Se La is the high-altitude mountain pass located in Tawang District of Arunachal
Pradesh.
 It has an elevation of 4170 m (13,700 ft) connects the Buddhist city of Tawang Town to Tezpur and
Guwahati.
 It is the main road connecting Tawang with the rest of India.
 The Sela Lake also known as Paradise Lake is located near to the pass.
 It is usually open throughout the year unless landslides or snow require the pass to be shut down
temporarily.

Q 5. (C)
 The National Commission for Safai Karamcharis (NCSK) was established as a statutory body in the year
1994 as per the provisions of the NCSK Act 1993 initially for the period of 3 years.
1

 Later the validity of the Act was initially extended up to 31.3.2002 and thereafter up to 29.2.2004.
Page

 The NCSK Act ceased to have effect from 29.2.2004.


Test: 17 (2020) | Test Code: A-20117 Contact us at www.iasprayojan@gmail.com / +91-7558644556
FREE BOOKS, NOTES & VIDEOS FOR CIVILSERVICES

EBOOKS & UPSC PRELIMS USPC MAINS VIDEO FOR DAILY


MAGZINES MATERIALS MATERIALS CIVILSERVICES NEWSAPERS

SECUREIAS UPSC PRELIMS UPSC MAINS DELHI CIVILSERVICES


TESTSERIES TESTSERIES STUDENTS BOOKS

OPTIONAL SUBJECTS BOOKS, STATE PCS, SSC, BANKING


TEST SERIES, VIDEOS & NOTES BOOKS, TESTS VIDEOS & NOTES
1.GEOGRAPHY 1.UPPSC 2.SSC 3.MPSC
2.HISTORY 4.IBPS 5.RAS & RPSC
3.MATHEMATICS ENGINEERING BOOKS & MATERIAL
4. SOCIOLOGY 1. IES 2. GATE 3. IFoS
5.PUBLIC ADMINISTRATION 4. COMPUTER SCIENCE
6. POLITICAL SCIENCE 5. MECHINICAL ENGINEERING
7. ECONOMICS OTHER TELEGRAM CHANNELS
8 PHYSICS 1 GOVERNMENT JOBS
9 COMMERCE ACCOUNTANCY 2 LEARN YOGA & MEDITATION
10 ANTHROPOLOGY 3 LEARN ENGLISH
11 LAW 4 BEST DELAS & OFFERS
12 PHILOSOPHY 5 IAS HINDI BOOKS
13 CHARTERED ACCOUNTANTANCY 6 PDFs FOR ALL EXAMS
14 MEDICAL SCIENCE 7. WORLD DIGITAL LIBIRARY
1.CHENNAI STUDENTS 2.BANGLORE STUDENTS 3. CURRENT AFFAIRS
CONTACT FOR ADVERTISEMENT IN ABOVE CHANNLES
ADMIN1: ADMIN2:
www.iasprayojan.com IAS PRAYOJAN NAGPUR
 After that the tenure of the NCSK has been extended as a non-statutory body from time to time.
 The tenure of the present Commission is up to 31.3.2019.
 The Union Cabinet has recently approved the proposal for Extension of tenure of the National Commission
for Safai Karmacharis (NCSK) beyond 31.3.2019 for three years.

Q 6. (D)
 The Battle of Sinhagad was fought between Tanaji Malusare, a commander of Maratha ruler Shivaji
Maharaj and Udaybhan Rathod, fort keeper under Jai Singh I who was a Mughal Army Chief in 1670 on the
fort of Sinhagad near the city of Pune, Maharashtra, India.
 Tanaji Malusare, the brave Maratha warrior played a major role during the Battle of Sinhagad in 1670 AD.
 Malusare died in 1670 after being seriously wounded in the Sinhagad battle.
 It is Tanaji Malusare after whom Shivaji is said to have renamed Kondhana fort to Sinhagad Fort (Lion‘s
fort).

Q 7. (A)
 U.S has recently proposed the Green New Deal (GND) to address both climate change and economic
inequality.
 The Green New Deal ―is a four-part programme for moving America quickly out of crisis into a secure,
sustainable future.
 The resolution was introduced in the House and Senate recently.

Q 8. (D)
 Chowra is an 8 sq.km. Island on the Andaman sea, part of the Nicobar archipelago.
 And on it live 1,350 people, the Nicobarese islanders of Chowra.
 The grasslands were burnt every year by these islanders.
 This ritual, called issoh, helped regenerate the grass they needed to thatch their houses before the
monsoon.
 The 2004 tsunami washed away almost every traditional house on Chowra island.
 Further rehabilitation efforts have forced the islanders into nuclear housing units and devastated the
grasslands.
 As the grasslands diminished, so did the cultural practices like issoh diminished.

Q 9. (C)
 The Hindu Kush Himalaya (HKH) spread over 3,500 kilometres from Afghanistan in the west to Myanmar
in the east.
 The HKH is one of the greatest mountain systems in the world, covering eight countries: Afghanistan,
Bangladesh, Bhutan, China, India, Myanmar, Nepal and Pakistan.
 The glaciers feed 10 of the world's most important river systems, including the Ganges, Indus, Yellow,
Mekong and the Irrawaddy.

Q 10. (A)
 Kerala has become the first State to set up a price monitoring and research unit (PMRU) to track violation
of prices of essential drugs and medical devices under the Drugs Price Control Order (DPCO).
 After Kerala, it is now the turn of Punjab and Gujarat to have PMRU, set up by the National Pharmaceutical
Pricing Authority in collaboration with the Department of Health and State Drug Controller.
 The National Pharmaceutical Pricing Authority (NPPA) had proposed such a system for the States and the
Union Territories five years ago.
 PMRU is aimed at monitoring the notified prices of medicines, detection of violation of the provisions of
DPCO, pricing compliance and ensuring availability of medicines, among other objectives.
 PMRU‘s will function under the direct supervision of State drug controller.
2
Page

Test: 17 (2020) | Test Code: A-20117 Contact us at www.iasprayojan@gmail.com / +91-7558644556


www.iasprayojan.com IAS PRAYOJAN NAGPUR
Q 11. (B)
 Tagore Award for Cultural Harmony was instituted by the Government of India from 2012 recognizing the
contributions made by Gurudev Rabindranath Tagore to humanity at large with his works and ideas, as part
of the Commemoration of his 150thBirth Anniversary in 2012, for promoting values of Cultural Harmony.
 It is awarded annually and carries an amount of Rs. One Crore (convertible to foreign currency), a citation
in a Scroll, a Plaque as well as an exquisite traditional handicraft / handloom item.
 A committee which selects the awardee comprises of Prime Minister of India, Chief Justice of India, Leader
of Opposition and two eminent personalities.
 The Award is open to all persons regardless of nationality, race, language, caste, creed or gender.
 The President of India had recently presented the Tagore Award for Cultural Harmony for the years 2014,
2015 and 2016 to Shri Rajkumar Singhajit Singh, Chhayanaut (a cultural organization of Bangladesh) and Shri
Ram Vanji Sutar respectively.

Q 12. (B)
 Originally known as Victoria Diamond, Jacob diamond was found in Kimberly mines in South Africa in 1884
and was secretly transported to England to avoid heavy duties then in place for raw diamonds.
 It is almost double the size of Kohinoor Diamond (in the present shape).
 The finished gem, with 58 facets, weighing 185. 75 carats was stunningly beautiful, in its cut, clarity and
colour. (Kohinoor weighed only 105.6 carats).
 It is a part of Nizam of Hyderabad‘s jewellery collection which was purchased in 1995 by the Government
of India at a cost of Rs. 218 Crore.
 Recently, it was kept in the exhibition of rare collection of Nizam’s jewels and jewellery at National Museum,
New Delhi.

Q 13. (C)
GODAVARI RIVER:
• The Godavari is India’s second longest river after the Ganga. Its source is in Triambakeshwar, Maharashtra.
It flows east for 1,465 kilometers.
• The drainage basin of the river is present in following states of India: Chhattisgarh, Maharashtra, Andhra
Pradesh, Telangana, Madhya Pradesh, Karnataka, and Orissa.
TRIBUTARIES OF GODAVARI:
• Left Bank Tributaries: Dharna, Penganga, Wainganga, Wardha, Pranahita [conveying the combined waters
of Penganga, the Wardha and Wainganga], Pench, Kanhan, Sabari, Indravati etc.
• Right Bank Tributaries: Pravara, Mula, Manjra, Peddavagu, Maner etc.
POLAVARAM PROJECT:
• It is an under construction multi-purpose National project on the Godavari River in the West Godavari
District and East Godavari District in Andhra Pradesh.
• The project has been accorded national project status by the Union Government of India.
• The back water of its reservoir spreads into parts of Chhattisgarh and Odisha States.
• It gives major boost to tourism sector in Godavari Districts as the reservoir covers the famous Papikonda
National Park, Polavaram Hydroelectric project(HEP) and National waterway 4 are in under construction at
left side of the river.

Q 14. (A)
• Pair 3 is incorrectly matched: Valley offlowers is in Uttarakhand.
• Correct Table
(Famous Valley) (State)
1. Neelum Valley : Jammu andKashmir
2. Spiti Valley : HimachalPradesh
3. Valley of flowers : Uttarakhand
3
Page

Test: 17 (2020) | Test Code: A-20117 Contact us at www.iasprayojan@gmail.com / +91-7558644556


www.iasprayojan.com IAS PRAYOJAN NAGPUR
NEELUM VALLEY, SPITI VALLEY, VALLEY OFFLOWERS
• Neelum Valley is a Himalayan gorge in the Kashmir region, along which the Neelum River flows. This green
and fertile valley is one of the most attractive tourists places, like Swat and Chitral, but due to poor road
system is yet veiled to the outside world.
• Spiti Valley is a cold desert mountain valley located high in the Himalayas in the northeastern part of the
northern Indian state of Himachal Pradesh. The name “Spiti” means “The Middle Land”, i.e. the land between
Tibet and India. Along the northern route from Manali, Himachal Pradesh or Keylong via the Rohtang Pass
or Kunzum Pass respectively, the valley lies in the North Eastern section of the Indian state Himachal
Pradesh.
• Valley of Flowers National Park is an Indian national park located in the state of Uttarakhand and is known
for its meadows of endemic alpine flowers and the variety of flora. This richly diverse area is also home to
rare and endangered animals, including the Asiatic black bear, snow leopard, musk deer, brown bear, red
fox, and blue sheep. Birds found in the park include Himalayan monal pheasant and other high altitude birds.

Q 15. (B)
• The Central African Republic is a landlocked country in Central Africa.
• It is bordered by Chad to the north, Sudan to the northeast, South Sudan to the east, the Democratic
Republic of the Congo to the south, the Republic of the Congo tithe southwest and Cameroon to the west.
• The Democratic Republic of the Congo, or simply the Congo, is a country located in Central Africa.

Q 16. (C)
• Statement 2 in Incorrect: It benefits mainly to formal sector Insured Persons’(IP).
ATAL BIMIT VYAKTI KALYAN YOJANA
• It is to financially support those who lost their jobs and were covered under the Employees’ State
Insurance Act, 1948.
• It benefits mainly to formal sector Insured Persons’ (IP).
• It was launched by ‘Employees’ State Insurance (ESI) Corporation’ under ‘Ministry of Labour &
Employment’.
• The cash benefit will be of 25% of the average per day earning during the previous four contribution
periods. • It is payable in case of unemployment ordering a search for new engagement.
• The scheme will be funded by the employees themselves from their ESI contribution.
• It is to be paid up to maximum 90 days of unemployment once in lifetime.
• It will be applicable to all factories and establishments employing at least 10workers.
4
Page

Test: 17 (2020) | Test Code: A-20117 Contact us at www.iasprayojan@gmail.com / +91-7558644556


www.iasprayojan.com IAS PRAYOJAN NAGPUR
Q 17. (A)
Explanation:
• Statement 1 is incorrect: No RTI fee is required to be paid by a citizen who is below poverty line, as per
RTI Rules, 2012
• Statement 2 is incorrect: The CIC have the power to order inquiry into any matter, suo moto,if there are
reasonable grounds.
RIGHT TO INFORMATION
• The Right to Information is embedded in Article 19. Consequently, the Right to Information (RTI) Act was
operationalized in October 2005.
• As per the RTI Act 2005, every citizen has the right to receive a timely response from the government for
any information that is sought by them with respect to the functioning of the government.
• An RTI portal is created by the Ministry of Personnel, Public Grievances and Pensions in order to facilitate
the process of RTI.
• Besides access to information related to RTI, published disclosures by various public authorities under the
state and the central governments, it acts as a gateway for obtaining information on the details of first
Appellate Authorities, Principle Information Officers etc.
CENTRAL INFORMATON COMMISSION
• The Central Information Commission was established by the Central Government in2005.
• It is not a constitutional body.
• The Commission consists of a Chief Information Commissioner and not more than ten Information
Commissioners.
• They are appointed by the President on the recommendation of a committee consisting of the Prime
Minister as Chairperson, the Leader of Opposition in the Lok Sabha and a Union Cabinet Minister nominated
by the Prime Minister
• The Chief Information Commissioner and an Information Commissioner hold office for a term of 5 years or
until they attain the age of 65 years, whichever is earlier.
• They are not eligible for reappointment.
Q 18. (C)
PLOONETS
• When the moons of exoplanets breakaway from their own orbits, went rogue and acts like a planet, it is
called “Ploonet.”
• It gets it name from ‘Planet + moon =Ploonet’.
• As the exoplanets move inward toward their suns, the orbits of their moons are often disrupted.
• So the moon may run away from their exoplanets and could become ‘Ploonets’, according to new study
models.
• This is because of the combined gravitational forces of the planet and the star.
• This gravitational force would inject extra energy into the moon’s orbit, pushing it farther from its planet
until eventually it escapes.
• This process happens in every planetary system composed of a giant planet in a very close-in orbit.
• As for Earth’s own Moon, it is a potential ploonet.
• It moves about 4 cm farther away from Earth every year.
• Going at this rate, it won’t break away from the Earth’s orbit for about next 5 billion years.
• However, astronomers not yet confirmed the existence of a single exomoon, it just remains hypothetical
in research papers.
• Criteria to classify any object as a “Planet”, according to the International Astronomical Union are, – It
needs to be in orbit around any full-fledged star.
– It needs to have enough gravity to pull itself into a spherical shape.
– It has cleared the neighborhood around its orbit.
5
Page

Test: 17 (2020) | Test Code: A-20117 Contact us at www.iasprayojan@gmail.com / +91-7558644556


www.iasprayojan.com IAS PRAYOJAN NAGPUR
Q 19. (B)
• Statement 1 is incorrect: The Court may entertain two types of cases: legal disputes between States
submitted to it by them (contentious cases) and requests for advisory opinions on legal questions referred
to it by United Nations organs and specialized agencies (advisory proceedings).
• Statement 3 is incorrect: Justice Dalveer Bhandari from India currently serves as permanent judge at ICJ.
International Court of Justice (ICJ)
• ICJ is the primary judicial branch of the United Nations.
• Seated in the Peace Palace in The Hague, Netherlands, the court settles legal disputes submitted to it by
states and provides advisory opinions on legal questions submitted to it by duly authorized international
branches, agencies, and the UN General Assembly.
• It is composed of 15 judges elected to 9year terms and may be re-elected for up to2 further terms.
• It has two primary functions: to settle legal disputes submitted by States in accordance with established
international laws, and to act as an advisory board on issues submitted to it by authorized international
organizations.
• In Contentious Cases, the judgment is final, binding on the parties to a case and without appeal
• In case of Advisory proceedings, the opinions by the Court do not have binding force.

Q 20. (A)
• Statement 2 is incorrect: Well performing institutions of India will mentor those Higher Education
Institutions which aspire to get the accreditation from National Accreditation and Assessment Council
(NAAC). (The statement in the questions related to Global Initiative of Academic Networks (GIAN))
PARAMARSH SCHEME
• The scheme aims for mentoring the National Accreditation and Assessment Council (NAAC) Accreditation
Aspirant Institutions to promote Quality Assurance in Higher Education.
• The scheme will be a paradigm shift in the concept of mentoring of institution by another well performing
institution to upgrade their academic performance and enable them to get accredited by focusing in the area
of curricular aspects, teaching learning& evaluation, research, innovation, institutional values & practices
etc.
• The scheme will be operationalized through “Hub & Spoke” model wherein the Mentor Institution, called
the “Hub” is centralized and will have the responsibility of guiding the Mentee institution through the
secondary branches the “Spoke” i.e. Through the services provided to the mentee for self-improvement.
• The scheme will target 1000 Higher Education Institutions (HEIs) for mentoring with a specific focus on
quality as enumerated in the UGC “Quality Mandate”.
• The scheme is expected to have a major impact in addressing a national challenge of improving the quality
of Higher Education in India.
• It will also facilitate sharing of knowledge, information and opportunities for research collaboration and
faculty development in Mentee Institutions.
• It will target 1000 Higher Education Institutions for mentoring with a specific focus on quality as
enumerated in the UGC “Quality Mandate”.
• The scheme also proposes to provide financial assistance to the mentoring institutions and the option of
appointing an expert, who can be paid a fellowship amount of Rs. 31,000 per month.
Q 21. (B)
• Statement 1 is incorrect: Chandipura Virus is known to cause inflammation of the brain in humans and
progresses rapidly from an influenza-like illness to coma and death.
Chandipura Virus
• The virus spreads mainly through the bite of sand flies, and sometimes through mosquitoes.
• Animal studies show that the virus affects neurons and causes neurodegeneration.
• Sand flies, which are found in mud and in cracks of sand houses, mostly breed during monsoon and pre-
monsoon months which is when the cases are generally reported.
• The likely vector (or carrier) of the virus is the female phlebotomine sand-fly. The virus was detected in
6

sandflies in Senegal, Nigeria as well as in India.


Page

Test: 17 (2020) | Test Code: A-20117 Contact us at www.iasprayojan@gmail.com / +91-7558644556


www.iasprayojan.com IAS PRAYOJAN NAGPUR
• CHPV belongs to the Rhabdoviridae family in the order Mononega virales ofthe genus Vesiculo virus.
• Interestingly, its continuing mutating trend has enhanced its lethality to cause human infections, unlike its
genetic cousin, the vesicular stomatitis virus (VSV).
• The virus predominantly infects children between the ages of 2-16, spreading through the bite of a sand-
fly, and in some cases, even the mosquito during the monsoon and pre-monsoon season.
• It is distantly related to the virus that causes rabies and is known to have a case fatality between 55-75%.

Q 22. (D)
Honey Bee Mission
• ‘Honey Mission’ was launched by Khadi and Village Industries Commission (KVIC) in August 2017 to attain
sweet revolution in India.
• It aims to provide beekeepers
• practical training about examination of honeybee colonies
• acquaintance with apicultural equipment’s
• Identification and management of bee enemies and diseases
• Honey extraction and wax purification
• Management of bee colonies in spring, summer, monsoon, autumn and winter seasons
Sweet Revolution
• Sweet Revolution is a strategic step to emphasize the increase in honey production in the state, which can
be a major contributor to doubling the income of the farmers.
• It was launched first in Jharkhand by its state agriculture department to –
• To make Jharkhand state in the category of developed states by 2022.
• Providing employment by connecting the farmers of the state with bee keeping.
• To increase agriculture and horticulture yield and income of farmers.
• To make the state the leading state in the field of honey production.
• Children free from malnutrition.
• Production of quality honey.

Q 23. (C)
Correct Order:
2. Guru Nanak(1st Guru)
1. Guru Angad (2nd Guru)
4. Guru Ram Das (4th Guru)
2. Guru Teg Bahadur (9th Guru)

SIKH GURUS1. Guru Nanak Dev - Guru from 1469 to 1539


• Guru Nanak Dev, first of the 10 gurus, founded the Sikh faith, introducing theconcept of one God.
• He started the institution of Guru KaLangar.
2. Guru Angad Dev - Guru from 1539 to1552
• He introduced the Gurmukhi (written form of Punjabi) script.
• He compiled the writings of Nanak Devin Guru Granth Sahib in Gurmukhi Script.
3. Guru Amardas Sahib - Guru from 1552to 1574
• Guru Amardas introduced the AnandKaraj marriage ceremony for the Sikhs, replacing the Hindu form.
• He established Manji & Piri system of religious missions for men and women respectively.
• He also completely abolished amongst the Sikhs, the custom of Sati and purdah system.
4. Guru Ram Das - Guru from 1574 to 1581
• He founded the city of Amritsar.
• He started the construction of the famous Golden Temple at Amritsar, the holy city of the Sikhs.
• He requested the Muslim Sufi, Mian Mirto lay the cornerstone of the Harmandir Sahib.
5. Guru Arjan Dev - Guru from 1581 to 1606
7

• He compiled the Adi Granth, the scriptures of the Sikhs.


Page

Test: 17 (2020) | Test Code: A-20117 Contact us at www.iasprayojan@gmail.com / +91-7558644556


www.iasprayojan.com IAS PRAYOJAN NAGPUR
• He completed construction of Sri Darbar Sahib also known as Golden Temple in Amritsar.
6. Guru Har Gobind Sahib - Guru from 1606to 1644
• He organized a small army and became the first Guru to take up arms to defend the faith.
• He waged wars against Mughal rulers Jahangir and Shah Jahan.
7. Guru Har Rai Sahib - Guru from 1644 to1661
• Though he was a man of peace, he never disbanded the armed Sikh warriors who were earlier maintained
by Guru Har Gobind.
8. Guru Har Krishan Sahib - Guru from 1661to 1664
• Guru Har Krishan was the youngest of the Gurus. He was installed as Guru at the age of five.
9. Guru Tegh Bahadur Sahib - Guru from1665 to 1675
• He established the town of Anandpur.
• He opposed the forced conversion of the
Hindu Kashmiri Pandits by Mughal rulers
10. Guru Gobind Singh Sahib - Guru from1675 to 1708
• He became Guru after the martyrdom of his father Guru Tegh Bahadur.
• He created the Khalsa in 1699, changing the Sikhs into a saint-soldier order for
protecting themselves.
• Last Sikh Guru in human form and he passed the Guru ship of the Sikhs to the Guru Granth Sahib.
11. Guru Granth Sahib
• Guru Granth Sahib (also known as the Adi Granth) is the scripture of the Sikhs.
• The Granth was written in Gurmukhi script and it contains the actual words and verses as uttered by the
Sikh Gurus.
• It is considered the Supreme Spiritual Authority and Head of the Sikh religion, rather than any living person.
KARTARPUR GURUDWARA
• Gurdwara Darbar Sahib Kartarpur, also called Kartarpur Sahib, is a gurdwara inKartarpur Pakistan.
• It is built on the historic site where Guru Nanak settled and assembled the Sikh commune after his
missionary travels.
• The present gurdwara is built on the sitewhere Guru Nanak Dev Ji died on 22September 1539.

Q 24. (C)
Option (c) is correct: One must seek shelter inside a car because the phenomenon of Electrostatic Shielding
will protect the person from the electric current.
How Lightning Strikes?
Lightning – It is a very rapid and massive discharge of electricity in the atmosphere, some of which is directed
towards the Earth’s surface.
Process
• As water vapor moves upward in the cloud, the falling temperature causes it to condense. Heat is
generated in the process, which pushes the molecules of water further up.
• As they move to temperatures below zero degrees Celsius, the water droplets change into small ice
crystals. They continue to move up, gathering mass until they are so heavy that they start to fall to Earth.
• This leads to a system in which, simultaneously, smaller ice crystals are moving up and bigger crystals are
coming down.
• Collisions follow, and trigger the release of electrons a process that is very similar tithe generation of sparks
of electricity. As the moving free electrons cause more collisions and more electrons, a chain reaction ensues.
• The electrical potential difference between the two layers is huge — of the order of a billion to 10 billion
volts. In very little time, a massive current, of the order of 100,000to a million amperes, starts to flow
between the layers.
How does this current reach the Earth from the cloud?
• While the Earth is a good conductor of electricity, it is electrically neutral. However, in comparison to the
middle layer of the cloud, it becomes positively charged.
8

• As a result, about 15%-20% of the current gets directed towards the Earth as well.
Page

Don’ts
Test: 17 (2020) | Test Code: A-20117 Contact us at www.iasprayojan@gmail.com / +91-7558644556
www.iasprayojan.com IAS PRAYOJAN NAGPUR
• There is a greater probability of lightning striking tall objects such as trees, towers or buildings because
lightning tends to change course towards these taller objects
• People are most commonly struck by what are called ground currents (electrical energy, after hitting a
large object spreads laterally on the ground for some distance). So, one should avoid lying flat on the
ground.
• Since water is good conductor of electricity, so one should stay away from any nearby water body like
ponds, lakes or swimming pool.
Do’s
• Seek shelter inside a car because the phenomenon of Electrostatic Shielding will protect the person from
the electric current.
• If you are out in the open, then crouch down and make sure that you are in minimum contact with the
ground. This will control the possible damage by the lighting during the thunderstorm.
Electrostatic Shielding
• Electrostatic shielding is defined as a phenomenon that is seen when a Faraday cage is used to block the
effects of an electric field.
• The effects of external fields on the internal contents are blocked using the cage.
• The metallic body of the car acts as electrostatic shielding from lightening.

Q 25. (D)
• Statement 1 is incorrect: Dengue virus is transmitted by female mosquitoes mainly of the species Aedes
aegypti and, to a lesser extent, Ae. albopictus.
• Statement 2 is incorrect: Recovery from dengue infection by one serotype (out of 4) provides lifelong
immunity against that particular serotype. However, cross immunity to the other serotypes after recovery is
only partial and temporary.
• Statement 3 is incorrect: Dengvaxia Vaccine is manufactured by Sanofi Pasteurs, a pharmaceutical
company in France.
Dengue Disease
• It is a mosquito-borne viral infection.
• The infection causes flu-like illness, and occasionally develops into a potentially lethal complication called
severe dengue.
• It is found in tropical and sub-tropical climates worldwide, mostly in urban and semi-urban areas.
• There is no specific treatment for dengue/ severe dengue, but early detection and access to proper
medical care lowers fatality rates below 1%.
• Dengue virus is transmitted by female mosquitoes mainly of the species Aedesaegypti and, to a lesser
extent, Ae.albopictus. This mosquito also transmits chikungunya, yellow fever and Zika infection.
• It is caused by a virus of the Flaviviridae family and there are 4 distinct, but closely related, serotypes of
the virus that cause dengue (DEN-1, DEN-2, DEN-3 and DEN-4).
• Recovery from infection by one provides lifelong immunity against that particular serotype. However,
cross-immunity to the other serotypes after recovery is only partial and temporary. Subsequent infections
(secondary infection) by other serotypes increase the risk of developing severe dengue.
Dengavaxia
• It is indicated for the prevention of dengue disease caused by dengue virus serotypes1, 2, 3 and 4.
• It is a live, attenuated dengue virus.
• An attenuated virus is a virus that retains its properties of triggering an immune response in the body but
its ability to lead to a disease is compromised.

Q 26. (A)
• Option (a) is correct: It has been launched by Central Industrial Security Force (CISF).
Securitypedia
• It is a website which incorporates a wide range of security related issues, best practices and latest
9

technologies (in the domain of security) available across the world.


Page

Test: 17 (2020) | Test Code: A-20117 Contact us at www.iasprayojan@gmail.com / +91-7558644556


www.iasprayojan.com IAS PRAYOJAN NAGPUR
• It is not just a static website containing data archives but is more of a dynamic platform where a force
member can contribute by writing blogs on relevant security related professional issues.
• Just like Wikipedia, any member could post information or edit articles after citing authentic and verifiable
sources.
• As a supplement to Securitypedia, CISF has also developed CISF Tube which is an online video database of
all videos relevant to CISF.
• The videos available on CISF Tube enables CISF personnel to learn on various subjects related to their area
of functioning.
Central Industrial Security Force (CISF)
• CISF is an armed force under the Union Home Ministry established under an Act of Parliament, Central
Industrial Security Force Act, 1968.
• It has 10 reserve battalions, 08 training institutes and 39 other organizations.
• According to the mandate, CISF provides security to the premises staff along with the security of property
and establishments.
• It is providing security to the strategic establishment, including the Department of Space, the Department
of Atomic Energy, the Airports, the Delhi Metro, the ports, the historical monuments and the basic areas of
Indian economy such as petroleum and natural gas, electricity, coal, steel and mining.
• It is providing protection to some private sector units and important government buildings in Delhi

Q 27. (D)
Mughal Coinage
• The standard gold coin of the Mughals was the Mohur of about 170 to 175 grains. Abul Fazl in his ‘Ain-i-
Akbari’ indicated that a Mohur was equivalent to nine rupees. Half and quarter mohurs are also known.
• The silver rupee which was an adoption from Sher Shah’s currency, was the most famous of all Mughal
coins.
• Akbar issued both round and square coins. In 1579, he issued gold coins called Ilahi coins to propagate his
new religious creed ‘Din-i-Illahi’. On this coin, it was written ‘God is great, may his glory be glorified’.
Sahansah was the largest gold coin. These coins bore the names of the persian solar months.
• Jahangir showed the legend in a couplet in the coins. In some of his coins, he added the name of his
beloved wife Noorjahan. The most famous of his coins had images of Zodiac signs.

Q 28. (C)
• Statement 1 is incorrect: Bitcoin is a type of digital currency. Fiat money is physical money—paper money
and coins.
• Statement 3 is incorrect: There are many methods to get bitcoins. Bitcoin Mining is just one of them.
Bitcoins
• Bitcoin is the first implementation of a concept called “cryptocurrency”. The first Bitcoin specification and
proof of concept was published in 2009 in a cryptography mailing list by Satoshi Nakamoto.
• It is a consensus network that enables a new payment system and completely digital money.
• It is the first decentralized peer-to-peer payment network that is powered by its users with no central
authority or middlemen.
• Nobody owns the Bitcoin network
Working of Bitcoin
• The Bitcoin network shares a public ledger called the “block chain”.
• This ledger contains every transaction ever processed, allowing a user’s computer to verify the validity of
each transaction.
• The authenticity of each transaction is protected by digital signatures corresponding to the sending
addresses, allowing all users to have full control overspending bitcoins from their own Bitcoin addresses.
• In addition, anyone can process transactions using the computing power of specialized hardware and earn
10

a reward in bitcoins for this service. This is often called “mining”.


Page

How does one acquire bitcoins?


Test: 17 (2020) | Test Code: A-20117 Contact us at www.iasprayojan@gmail.com / +91-7558644556
www.iasprayojan.com IAS PRAYOJAN NAGPUR
• As payment for goods or services.
• Purchase bitcoins at a Bitcoin exchange.
• Exchange bitcoins with someone near you.
• Earn bitcoins through competitive mining.

Q 29. (B)
• Option (b) is correct: Red Mud is residue of aluminum
Red Mud
• Red Mud is a solid waste generated during the aluminum production process.
• One ton of production of alumina leads to output of 1.5 ton of red mud.
• Global generation of red mud is more than150 million tons and there exists a global inventory of more
than 3 billion tons.
• Red mud generation in India is around 9million tons per year.
• Red mud contains impurities such as caustic soda and others minerals and thus it is a serious environmental
concern. This waste needs to be converted into wealth.
• Red mud contains 25 to 30 per cent of iron.

Q 30. (D)

Q 31. (C)
• Option (c) is correct: This pill is developed to assess gut bacteria throughout the gastrointestinal (GI) tract.
3D Printed Pill
Researchers have developed an ingestible 3Dprinted pill, which can non-invasively assess gut bacteria
throughout the gastrointestinal (GI) tract.
• This biocompatible pill is manufactured in a3D printer with microfluidic channels.
• It can easily sample different stages of the GI tract, which was till now impossible to track non-invasively.
• Current method is based on the use of DNA sequencing techniques to analyses bacteria found in the gut
11

known as the microbiome (The gut microbiome is comprised of the collective genome of microbes inhabiting
the gut including bacteria, archaea, viruses, and fungi).
Page

Test: 17 (2020) | Test Code: A-20117 Contact us at www.iasprayojan@gmail.com / +91-7558644556


www.iasprayojan.com IAS PRAYOJAN NAGPUR
Q 32. (A)
• Option (a) is correct: Maharashtra became the first state to adopt a digital fingerprint and iris scanning
system to aid police investigations namely Automated Multi-modal Biometric Identification System(or
AMBIS)
• Maharashtra became the first state to adopt a digital fingerprint and iris scanning system to aid police
investigations namely Automated Multi-modal Biometric Identification System (or AMBIS).
• An AMBIS unit comprises a computer terminal, a camera, and iris, fingerprint, and palm scanners. It also
includes a portable system to dust off and capture fingerprints from crime scenes. With the integration of
the system with facial recognition from CCTV cameras, AMBIS enables the police to cross-reference and put
faces to criminals whose fingerprints have been captured on paper over the decades, apart from solving
fresh crimes.
• AMBIS replaces the Automated Fingerprint
Identification System (AFIS), which has been used by Indian law enforcement agencies to search finger and
palm prints. However, AFIS has limited utility, providing only one to-one fingerprint matches as compared
multimodal matches possible with AMBIS. With facial recognition technology, the new system is also an
upgrade on AFIS.

Q 33. (B)
Internet Saathi Programme
• Google India’s ‘Internet Saathi’ programme aimed to empower rural women on how to use the Internet
and recently, it has now added two more states— Punjab and Odisha. With this, the programme has now
reached 2.6 lakh villages in 20 states.
• Launched as a pilot project in Rajasthan in July 2015, along with Tata Trusts, ‘Internet Saathi’ programme
focuses on educating women on how to use the Internet.
• These women, in turn, impart training to other women in their community and neighbouring villages.
programme led to a level of independence that creates financial opportunities for women to pursue their
dreams.

Q 34. (D)
 The National Tourism Advisory Council (NTAC) serves as a think tank of the Ministry of Tourism.
 The present NTAC was constituted on 20th October, 2016 under the Chairmanship of the Minister of
Tourism with tenure of 3 years.
 The committee constitutes important Ministries, individual experts in the field of travel and tourism
management and Ex-offico members from the industry associations.

Q 35. (B)

Q 36. (A)
 Yemen's government and Houthi rebels have agreed on phase one of a mutual pullback of forces from the
key city of Hodeidah.
 The deal on the first phase has been finalised and they have agreed in principle for the second phase.
 Under phase one, forces from both sides will withdraw from the ports of Hodeidah, Saleef, and Ras Issa.
 A pullback from Hodeidah was part of December's ceasefire between Iranian-backed Houthi rebels and the
Saudi-supported Yemeni forces.
12
Page

Test: 17 (2020) | Test Code: A-20117 Contact us at www.iasprayojan@gmail.com / +91-7558644556


www.iasprayojan.com IAS PRAYOJAN NAGPUR

Q 37. (B)
 As per the Indus Waters Treaty, signed between India and Pakistan in 1960, India has full rights to utilise
the waters of the Ravi, the Beas and the Satluj rivers, while the waters of the Indus, the Chenab and the
Jhelum belongs to Pakistan.
 A dam was started construction at Shahpur-Kanti on the River Ravi.
 The project aims to minimise the wastage of water that now goes to Pakistan through the Madhopur
Headworks downstream.
 The Ujh Project in Kathua, Jammu and Kashmir, aims to irrigate about 30,000 hectares and produce 300
MW electricity from the waters of the Ujh, a tributary of the Ravi.

Q 38. (D)
 The Insurance Regulatory and Development Authority of India (IRDAI) has mandated hospitals to ensure a
quality healthcare ecosystem through National Accreditation Board for Hospitals and Healthcare
Organizations (NABH) Entry-Level Certification Process.
 In order to extend benefits associated with IRDAI and Ayushman Bharat scheme to small scale hospitals
across India, the government has digitalized and simplified the NABH accreditation process.
 The revised process is driven through a new portal called HOPE - Healthcare Organizations‟ Platform for
entry-level-certification.
 It focuses to promote quality at nascent stages by enrolling a wide range of hospitals across the country
including healthcare organizations (HCOs) and small healthcare organizations (SHCOs).

Q 39. (C)
 Padayani is a ritual theatre art form related to the Goddess temples in the Central Travancore regions of
Kerala.
 Performed at night in the temple premises, the rituals of Padayani are a symbolic act for pleasing the
Goddess Bhadrakali whose fury remains unabated even after her victory over the mythical demon, Darikan.

Q 40. (A)
 River Congo is the only river to cross the equator twice.
 The Galapagos Islands, part of the Republic of Ecuador, are an archipelago of volcanic islands distributed
on either side of the equator in the Pacific Ocean.

Q 41. (B)
13

 The Maha Upanishad is a Sanskrit text and is one of the minor Upanishads of Hinduism.
Page

The text is classified as a Vaishnava Upanishad.


Test: 17 (2020) | Test Code: A-20117 Contact us at www.iasprayojan@gmail.com / +91-7558644556
www.iasprayojan.com IAS PRAYOJAN NAGPUR
 It is notable for its teaching of “Vasudhaiva Kutumbakam”, or ―the world is one family‖.

Q 42. (D)
 SHREYAS – Scheme for Higher Education Youth for Apprenticeship and Skills.
 SHREYAS is a programme conceived for students in degree courses, primarily non-technical, with a view
to introduce employable skills into their learning, promote apprenticeship as integral to education and also
amalgamate employment facilitating efforts of the Government into the education
system so that clear pathways towards employment opportunities are available to students during and after
their graduation.
 SHREYAS is a programme basket comprising the initiatives of three Central Ministries, namely the Ministry
of Human Resource Development, Ministry of Skill Development & Entrepreneurship and the Ministry of
Labour& Employment viz the National Apprenticeship Promotion Scheme (NAPS), the National Career
Service (NCS) and introduction of BA/BSc/BCom (Professional) courses in the higher educational institutions.

Q 43. (A)
 Ministry of Civil Aviation has launched the document VISION2040 at the Global Aviation Summit 2019
that held recently.

Q 44. (B)
 Formjacking is a new hacking technique that targets online shoppers.
 Formjacking attacks are essentially a virtual ATM skimming technique under which cyber criminals target
website of a retailer by injecting malicious codes.
 These codes give away access to online shoppers‘ payment details including sensitive card details.

Q 45. (D)
 Global Digital Health Partnership Summit is a global intergovernmental meeting on digital health.
 GDHP aims to improve the health and well-being of citizens through the best use of evidence-based digital
technologies.
 It is being hosted by the Ministry of Health and Family Welfare in collaboration with World Health
Organization (WHO) and the Global Digital Health Partnership (GDHP).

Q 46. (D)
 The Global Digital Health Partnership (GDHP) is an international collaboration of governments, government
agencies and multinational organisations dedicated to
improving the health and well-being of their citizens through the best use of evidence-based digital
technologies.
 Governments are making significant investments to harness the power of technology and foster innovation
and public-private partnerships that support high quality, sustainable health and care for all.
 The GDHP facilitates global collaboration and co-operation in the implementation of digital health services.

Q 47. (A)
 Nirbhay is India‘s first indigenously designed long-range sub-sonic cruise missile.
 Equivalent to the famous American Tomahawk missile, Nirbhay missile is a two-stage missile which carries
nuclear warheads of up to 300 kg at a speed of 0.6 -0.7 Mach.
 India had developed the Brahmos supersonic cruise missile with Russia but it had to go alone for developing
the Nirbhay, which has a range of 1,000 kilometres.
 This is because of the Missile Technology Control Regime (MTCR), which forbids its signatory countries
from assisting or providing technology to any other country developing a cruise missile with a range of 300
kilometres or more.
14

 The range of Brahmos has thus been capped at 295 kilometres, just under the limit set by MTCR.
Page

Test: 17 (2020) | Test Code: A-20117 Contact us at www.iasprayojan@gmail.com / +91-7558644556


www.iasprayojan.com IAS PRAYOJAN NAGPUR
Q 48. (B)
 It runs between India and Nepal

Q 49. (C)
The Tokyo 2020 Medal Project sparked the collection of nearly 80,000 tons of mobile phones and small
electronic devices around Japan, which will be used in the crafting of every gold, silver and bronze Olympic
and Paralympic medal awarded to athletes at next year's Olympic games.
Q 50. (D)

Q 51. (B)
 It is located in Andaman

Q 52. (C)
 Attukal Pongala is one of the largest religious congregations of women.
 Preparing ‗pongala‘ (a sweet offering) is considered an auspicious all-women ritual as part of the annual
festival of the Attukal Bhagavathy Temple, which is popularly known as the ―Women‘s Sabarimala‖.
 As per the locals, the Pongala festival commemorates the hospitality accorded by women in the locality to
Kannagi, the heroine of the Tamil epic Silappadhikaram while she was on her way to Kodungallur in Kerala,
after destroying Madurai city to avenge the injustice to her husband Kovalan.
 In 2009, the Pongala ritual had made it to the Guinness Book of World Records for being the largest
religious gathering of women on a single day when over 25 lakh women took part.

Q 53. (C)
 The Food Safety and Standards Authority of India (FSSAI) has launched the Repurpose Cooking Oil (RUCO)
initiative to collect and convert used cooking oil into bio-fuel.
 Recently, Dehradun-based Indian Institute of Petroleum has successfully finished a pilot test to convert used
cooking oil into bio-aviation turbine fuel (Bio-ATF), which can be blended with conventional ATF and used as
aircraft fuel.

Q 54. (C)
Sir Sayyid Ahmad Khan
 In the rise of the separatist tendency along communal lines, Sayyid Ahmad Khan played an important
role.
 Though a great educationist and social reformer, Sayyid Ahmad Khan became towards the end of his life a
conservative in politics.
 He laid the foundations of Muslim communalism when in the 1880’s he gave up his earlier views and
declared that the political interests of Hindus and Muslims were not the same but different and even
divergent.  He also preached complete obedience
to British rule. When the Indian National Congress was founded in 1885, he decided to oppose it and tried
to organize along with Raja Shiva Prasad of Varanasi a movement of loyally to British rule.
 He also began to preach that, since the Hindus formed the larger part of the Indian population, they would
dominate the Muslims in case of the weakening or withdrawal of British rule. He urged the Muslims not to
listen to Badruddin Tyabji’s appeal to them to join the National Congress.

Q 55. (B)
Shankaradeva
 In the late 15th century, Shankaradeva emerged as one of the leading proponents of Vaishnavism in Assam.
 His teachings, often known as the Bhagavati dharma because they were based on the Bhagvata Gita and
15

Bhagvata Purana, focused on absolute surrender to the supreme deity (i.e. Vishnu).
Page

Test: 17 (2020) | Test Code: A-20117 Contact us at www.iasprayojan@gmail.com / +91-7558644556


www.iasprayojan.com IAS PRAYOJAN NAGPUR
 He emphasized the need for naam kirtan, recitation of name of Lord in satsanga or congregations of pious
devotees.
 He also encouraged the establishment of satra or monasteries for the transmission of spiritual knowledge,
and naam ghar or prayer halls.
 His major compositions include the Kirtanaghosha.

Q 56. (D)
 Statement 1 incorrect: Sufism was initiated as a result of protest against the growing materialism of the
Caliphate.
 Statement 3 is incorrect: The Sufis organized communities around a hospice called ‘khanqah’.
 Statement 4 is incorrect: Chishti order was named after their place of origin – the town of Chist in Central
Afghanistan.
Growth of Sufism
 In the early centuries of Islam a group of religious minded people called Sufis turned to asceticism and
mysticism in protest against the growing materialism of the Caliphate as a religious and political
institution.
 They were critical of the dogmatic definitions and scholastic methods of interpreting the Qur’an and Sunna
(traditions of the Prophet) adopted by theologians. Instead, they laid emphasis on seeking salvation through
intense devotion and love for God by following His commands, and by following the example of the Prophet
Muhammad whom they regarded as a perfect human being.
 The Sufis thus sought an interpretation of the Quran on the basis of their personal experience.
 By the eleventh century Sufism evolved into a well-developed movement with a body of literature Quranic
studies and Sufi practices.
 The Sufis began to organize communities around the hospice or khanqah (Persian) controlled by a
teaching master known as shaikh (in Arabic), pir or murshid (in Persian). He enrolled disciples (murids) and
appointed a successor (Khalifa). He established rules for spiritual conduct and interaction between inmates
as well as between laypersons and the master.
 The word ‘silsila’ literally means a chain, signifying a continuous link between master and disciple,
stretching as an unbroken spiritual genealogy to the Prophet Muhammad.
 When the Sufi teacher Shaikh died, his tomb-shrine (dargah, a Persian term meaning court) became the
centre of devotion for his followers. This encouraged the practice of pilgrimage or ziyarat to his grave,
particularly on his death anniversary or urs (or marriage, signifying the union of his soul with God).
 Most Sufi lineages were named after a founding figure. For instance, Qadri order was named after Shaikh
Abdul Qadir Jilani. However the Chisti order was named after their place of origin – the town of Chist in
Central Afghanistan.

Q 57. (A)
 Pairs 2 and 3 are incorrectly matched:
Author/Artists Mughal Emperor
1. Abul Fazl Akbar
2. Abdul Hamid Lahori Shahjahan
3. Abdus Samad Humayun
Author/Artists in the Mughal Court
 Abdul Hamid Lahori, Abul Fazl and Abdus Samad were present in the courts of Shahjahan, Akbar and
Humayun respectively.

Q 58. (D)
 Option (d) is correct: The correct order of Mahajanpadas from north to south is:
Taxila-Koshala-Magadha-Avanti.
16

Supplementary notes:
Page

The age of Mahajanapadas


Test: 17 (2020) | Test Code: A-20117 Contact us at www.iasprayojan@gmail.com / +91-7558644556
www.iasprayojan.com IAS PRAYOJAN NAGPUR
 The sixth-century B.C was a period of intense and new political development besides socio-political and
religious upheavals that brought two religions to the fore-Buddhism and Jainism. In the later Vedic age,
agriculture and iron tools helped people to settle down at one particular place.
 The permanent settlement led to the foundation of the Jana padas or small territorial states under the
control of a king. The main area of political activity gradually shifted from Western UP to eastern Upland
Bihar. This region was not only fertile due to the rainfall and river systems but also closer to iron production
centers. These of better iron tools and weapons along with sound economic growth resulted in the
transformation of some territorial states into Bigger and more powerful states, which came to be known as
Mahajanapdas.
 There were 16 Mahajanapadas in ancient
India. They were:
 Anga
 Magadha
 Kasi
 Vatsa
 Kosala
 Saurasena
 Kuru
 Matsya
 Chedi
 Avanti
 Gandhara
 Kamboja
 Ashmaka
 Vajji
 Malla

Q 59. (A)
 Statement 3 is incorrect: Duties on internal trade were abolished by MirQasim after the Battle of Plassey.
 Statement 4 is incorrect: Mir Qasim shifted the capital from Murshidabad to Munger to allow a safe
distance from the company.
Battle of Plassey
 The beginnings of British political sway over India may be traced to the battle of Plassey in 1757, when the
English East India Company’s forces defeated Siraj-ud-Daulah, the Nawab of Bengal.
 Causes of the battle:
 The Company had secured valuable privileges in 1717 ‘under a royal Farman by the Mughal Emperor, which
had granted the Company the freedom to export and import their goods in Bengal without paying taxes and
the right to issue passes or dastaks for the movement of such goods. The Company’s servants were also
permitted to trade but were not covered by this Farman and were required to pay the same taxes as Indian
merchants.
 This Farman was a perpetual source of conflict between the Company and the Nawabs of Bengal. For one,
it meant loss of revenue to the Bengal Government.
‘Secondly, the power to issue dastaks for the Company’s goods was misused by the Company’s servants
to evade taxes on their private trade.
 Without taking the Nawab’s permission, the Company began to fortify Calcutta in expectation of the
coming struggle with the French, who were stationed at this time at Chandernagore.
 Siraj·ud·Daulah interpreted this as an attack on his sovereignty and ordered both the English and French
to demolish their fortifications.
 While the French obliged, English refused to do so. This set the stage fora battle which took place on the
17

field of Plassey on 23rd June, 1757.


Page

Test: 17 (2020) | Test Code: A-20117 Contact us at www.iasprayojan@gmail.com / +91-7558644556


www.iasprayojan.com IAS PRAYOJAN NAGPUR
 The fateful battle of Plassey was a battle only in name. In all, the English lost 29men while the Nawab lost
nearly 500. The major part of the Nawab’s army, led by the traitors Mir Jafar and Rai Durlabh, took no part
in the fighting.
 After the battle, Mir Jaffar was proclaimed the Nawab of Bengal and the company was granted undisputed
right to free trade in Bengal, Bihar and Orissa. It also received the zamindari of the 24 Parganas near Calcutta.

Q 60. (C)
 Option (c) is correct: Cholas undertook the famous Kadaram campaign in AD 1025-26.
Kadaram Campign
 The Cholas were well known for their sustained naval policy from the days of rajaraja to kulottunga1, a
rare feature in early Indian polity and politics. Rajaraja initiated the process by sending fleets toIlam or Sri
Lanka, the northern part of which came under the Chola rule. The chola fleet became stronger during
Rajenndra1’stime when the entire island of Ilam was conquered.
 This campaign was followed by the most daring Chola raids, the kadaram campaign in AD1025-26 When
no less than tweleve areas in south-east asia were conquered by the Chola fleet.

Q 61. (C)
 Statement 2 is incorrect: To recognize Manipur as an independent state.
Treaty of Yandabo
 The first Burmese war was officially declared on 24 February 1824. After an initial set-back, the British
forces drove the Burmese out of Assam, Cachar, Manipur,and Arakan. The British expeditionary forces by
sea occupied Rangoon in May 1824and reached within 45 miles of the capital at Ava.
 The famous Burmese General Maha Bandula was killed in April 1825. But Burmese resistance was tough
and determined. Especially effective was guerrilla warfare in the jungles.
 The rainy climate and virulent diseases added to the cruelty of the war. Fever and dysentery killed more
people than the war. In Rangoon 3,160 died in hospitals and 166on the battlefield. In all the British lost15,000
soldiers out of the 40,000 they had landed In Burma. Moreover, the war was proving financially extremely
costly.
 Thus the British, who were winning the war, as well as the Burmese, who were losing it, were glad to make
peace which came in February 1826 with the Treaty of Yandabo.
 The Government of Burma agreed:(1) to pay one crore rupees as war compensation; (2) ‘to cede its
coastal provinces of Arakan and Tenasserim; (3)to abandon all claims to Assam, Cachar, and Jaintia; (4) to
recognise Manipur as an independent state; (5) to negotiate a commercial treaty with Britain; (6)and to
accept a British Resident at Ava while posting a Burmese envoy at Calcutta. By this treaty, the British
deprived Burma of most of its coastline and acquired a firm base in Burma for future expansion.

Q 62. (A)
 Statement 2 is incorrect: There were also oligarchical mahajanapadas where power was exercised by a
group of people.
 Statement 3 is incorrect: The term Jana and not Jana pada is known from the days of the Vedic literature.
The age of Mahajan padas
 The sixth-century B.C was a period of intense and new political development besides socio-political and
religious upheavals that brought two religions to the fore-Buddhism and Jainism. In the later Vedic age,
agriculture and iron tools helped people to settle down at one particular place.
 The permanent settlement led to the foundation of the Jana padas or small territorial states under the
control of a king. The main area of political activity gradually shifted from Western UP to eastern UP and
Bihar. This region was not only fertile due to the rainfall and river systems but also closer to iron production
centers. the use of better iron tools and weapons along with sound economic growth resulted in the
18

transformation of some territorial states into Bigger and more powerful states, which came to be known as
Mahajanapdas.
Page

Test: 17 (2020) | Test Code: A-20117 Contact us at www.iasprayojan@gmail.com / +91-7558644556


www.iasprayojan.com IAS PRAYOJAN NAGPUR
 There were 16 Mahajanapadas in ancient India.
 The formation of the state polity was not uniformly seen in the subcontinent and it was not present in
peninsular Indian or in the Ganga delta and in the northeastern part of the subcontinent.
 The majority of these states were monarchical but some were also republics, known as ganasangha.
Ganasangha had an oligarchic system for governance where the administration was headedby an elected
king who had a large council for his aid.
 The term janapada literally denotes an area where a group of people or a tribe/clan(jana) first set its
foot/feet (pada). The word Janapada, therefore, clearly implies a well defined and populated territory. The
term Jana is known from the days of the Vedic literature but the word janapada in the sense of territorial
entity seems to have gained currency only from the post-Vedic times.

Q 63. (A)
 Option (a) is correct: Nastaliq is a calligraphic style with long horizontal strokes.
Nastaliq
 Calligraphy is an art of handwriting. It is considered as a skill of great importance. It was practised using
different styles. Akbar’s favourite was the nastaliq, a fluid style with long horizontal strokes.
 It is written using a piece of trimmed reed with a tip of five to 10 mm called qalam, dipped in carbon ink
(siyahi). The nib of the qalam is usually split in the middle to facilitate the absorption of ink.

Q 64. (D)
 The coastal plains constitute the states of Tamil Nadu in east and Kerala on west. In Tamil Nadu, the rivers
are seasonal. As a result, the people of this region have depended more on tank irrigation since the early
times. However, Kaveri delta has been the major region of human attraction.
 It provided opportunity for the cultivation of rice and witnessed the flourishing of the Sangam culture in
the early historical period. The ports, such as Arikamedu and Kaveripattinam gave impetus to the Indo-
Roman trade in the early centuries of the Christian era. The Tamil region evolved a distinct linguistic and
cultural identity of its own.
 The Raichur doab, for its rice cultivation, has been known as the ‘rice-bowl’ of South India. It has been the
bone of contention between different kingdoms.

Q 65. (D)
 A semi-arid region is advantageous to the people for settlement purpose. For example, the Sind region,
having this type of climate in ancient period, resulted in the flourishing Harappa civilization.
 It also helped the growth of urban settlements. Similarly, the rise of Pataliputra and the importance of
Magadha in Bihar can also be explained in relation to its physical features and environment.
 Pataliputra was surrounded by the rivers, namely the Ganges, Son and Gandak, which provided natural
defence as well as internal communication. Moreover, the fertile Indo-Gangetic plains helped in the
maintenance of a strong population base.
 An area gifted with navigable rivers has well developed trade and communication networks. Our ancient
literature, like the Jatakas and other texts, mention many riverine routes in ancient India.
 Similarly, the coastal routes promote the long-distance trade with different countries. The mountain passes
are also very important in this context. For example, the Palghat Pass linked the east and west coasts and
thus, helped in the growth of Indo-Roman trade in ancient times.

Q 66. (D)
 The Vedic literature consists of the four Vedas – Rig, Yajur, Sama and Atharva. The Sama Veda is set to tune
for the purpose of chanting during sacrifice. It is called the book of chants and the origins of Indian music are
traced in it.
19

 Besides the Vedas, there are other sacred works, like the Brahmanas, the Upanishads, the Aranyaka’s and
the epics Ramayana and Mahabharata.
Page

Test: 17 (2020) | Test Code: A-20117 Contact us at www.iasprayojan@gmail.com / +91-7558644556


www.iasprayojan.com IAS PRAYOJAN NAGPUR
 The Brahmanas are the treatises relating to prayer and sacrificial ceremony. The Upanishads are
philosophical texts dealing with topics like the soul, the absolute, the origin of the world and the mysteries
of nature.
 The Aranyaka’s are called forest books and they deal with mysticism, rites, rituals and sacrifices.

Q 67. (D)
 In 305 B.C., Chandragupta Maurya marched against Selukas Niketar, who was Alexander’s General
controlling the north western India. Chandragupta Maurya defeated him and a treaty was signed.
 By this treaty, Selukas Niketar cededthe trans-Indus territories – namely Aria, Arakosia and Gedrosia – to
the Mauryan Empire. He also gave his daughter in marriage to the Mauryan prince. Chandragupta made a
gift of500 elephants to Selukas.
 Chandragupta Maurya was the founder of the Mauryan Empire. He, at a young age, captured Pataliputra
from the last ruler of the Nanda dynasty, Dhanananda. In this task, he was assisted by Kautilya, who was also
known as Chanakya or Vishnugupta wherein the latter then became his most trusted advisor.

Q 68. (B)
 The land revenue under the Maurya was normally fixed as one sixth of the produce.
 Samharta was the chief of the Revenue Department and was in charge of the collection of all revenues of
the empire. The revenues came from land, irrigation, customs, shop tax, ferry tax, forests, mines and
pastures, license fee from craftsmen, and fines collected in the law courts.

Q 69. (A)
 Tolkkappiyam (written by Tholkappiyar) deals with grammar and poetics.
 Tirukkural (authored by Thiruvalluvar) deals with philosophy and wide maxims.
 Silappadikaram (by Ilango) and Manimekalai (by Sattanar) are twin Tamil epics.
 Ramavataram is popularly referred toas Kamba Ramayanam and is a Tamil epic that was written by the
Tamil poet Kamban during the 12th century.

Q 70. (C)
 The Pallavas introduced the art of excavating temples from the rock. Infact, the Dravidian style of temple
architecture began with the Pallava rule.
 It was a gradual evolution, starting from the cave temples to monolithic rathas and culminated in structural
temples.
 The Kailashanath temple at Kanchi and the Shore temple at Mahabalipuram remain the finest examples of
the early structural temples of the Pallavas. The temple at Kanchi is the greatest architectural
master piece of the Pallava art.
 Pattadakal served as the capital of the Chalukya dynasty, who built the temples. The Papanatha temple
was built by them.
 Built by Raja Raja I in 1010 AD, the Brihadeshwara Temple is an ancient temple at Thanjavur in Tamil Nadu.
The temple is a part of the UNESCO World Heritage Site, known as the "Great Living Chola Temples".

Q 71. (C)
 Narasimhavarman I was also known as Mamalla, which means ‘great wrestler’. He wanted to avenge the
defeat of his father at the hands of Chalukyan ruler, Pulakesin II.
 His victory over Pulakesin II in the Battle of Manimangalam, near Kanchi, is mentioned in the Kuram
copperplates.
 The Pallava army, under General Paranjothi, pursued the retreating Chalukya army, entered the Chalukya
territory, and captured and destroyed the capital city of Vatapi. Narasimhavarman I assumed the title
20

‘Vatapikonda’.
 He was the founder of Mamallapuram and the monolithic rathas were erected during his reign.
Page

Test: 17 (2020) | Test Code: A-20117 Contact us at www.iasprayojan@gmail.com / +91-7558644556


www.iasprayojan.com IAS PRAYOJAN NAGPUR
Q 72. (A)
 The Guptas organized a system of provincial and local administration. The empire was divided into divisions
called bhukti and each bhukti was placed under the charge of anuparika.
 The bhuktis were divided into districts(vishayas), which were placed under the charge of a vishayapati. In
eastern India, the vishayas were divided into vithis, which again were sub-divided into villages.
 The village headman gained importance in the Gupta times, managing village affairs with the assistance of
elders.
 With the administration of a village or a small town, leading local elements were associated. No land
transactions could be affected without their consent.

Q 73. (A)
Tolkappiyam refers to the five-fold division of lands – Kurinji (hilly tracks), Mullai (pastoral), Marudam
(agricultural),Neydal (coastal) and Palai (desert).The people living in these five divisions had their respective
chief occupations, as well as gods for worship.
• Kurinji – chief deity was Murugan –chief occupation, hunting and honey collection.
• Mullai – chief deity Mayon (Vishnu) –chief occupation, cattle-rearing and dealing with dairy products.
• Marudam – chief deity Indira – chief occupation, agriculture.
• Neydal – chief deity Varunan – chief occupation fishing and salt manufacturing.
• Palai – chief deity Korravai – chief occupation robbery.

Q 74. (D)
 Vallabhacharya (1479-1531), was an Indian Telugu Philosopher, who founded the Krishna centered Pushti
sect of Vaishnavism in the Braj region of India. His philosophy is called Shuddha Advaita (Pure Non-dualism).
 Pushtimarga (Path of Grace) is a Vaishnav sect of Hinduism, founded by Vallabhacharya around 1500 AD.
It be lived that because the Lord is accessible only through His own grace. The Lord cannot be attained by a
given formula - He is attainable only if He wants to be attained!
 The sect established by him is unique in its facets of devotion to Krishna, especially his child manifestation,
and is enriched with the use of traditions, music and festivals.
 Ashtachhap was group of 8 poets established by Vallabhacharya who by their writings eulogized various
facets of life of Sri Krishna.
1. Kumbhandas
2. Surdas
3. Krishnadas (Shudra by caste)
4. Parmanand Das
5. Govindswami
6. Cheetswami
7. Nand Das
8. Chaturbhuj Das

Q 75. (D)
 Bhagat Pipa was a Rajput prince bornin Gagron, Jhalawar. He was a prince who renounced his throne in
the search of spiritual solace and said that one should search within for God. He said that one's inner self
has a relation with God. Bhagat Pipa shared same views as Guru Nanak. Bhagat Pipa’s hymns are included in
the Guru Granth Sahib. He settled in near Kutch, Gujarat.
 Ravi Das was a mystic saint poet during the 15th and 16th century. He came from a shudra family which
worked with dead animals’ skins to produce leather products and very much known in India as untouchables.
He was one of the disciples of bhakti saint poet Ramananda. His devotional songs are included in Guru Granth
Sahib. Panch Vani text of Dadupanthi tradition also includes numerous poems of Ravidas. He was born and
died in Varanasi, Uttar Pradesh.
21

 Dadu Dayal was a poet saint from Gujarat, India. He later moved to Naraina, near Jaipur, Rajasthan, where
Page

he gathered around himself a group of followers, forming a sect became known as the Dadupanth. Dadu’s
Test: 17 (2020) | Test Code: A-20117 Contact us at www.iasprayojan@gmail.com / +91-7558644556
www.iasprayojan.com IAS PRAYOJAN NAGPUR
composition in Braj languages which were recorded by his disciple Rajjab and are known as the Dadu
Anubhav Vani.
 Dadu believed that devotion to God should transcend religious or sectarian affiliation, and that devotees
should become non-sectarian or "Nipakh". He instructed additional 52disciples to set up ashrams,'Thambas'
around the region to spread the Lord's word. Five thambas are considered sacred by the followers; Naraiana,
Bhairanaji, Sambhar, Amer,and Karadala (Kalyanpura). Followers at these thambas later set up other places
of worship.

Q 76. (C)
 To further strengthen his position in dealing with the Mullahs, Akbar also issued a declaration or Mazhar,
which asserted that if there were conflicting views among those who were considered fit to interpret the
Quran, that is Mujtaddids, Akbar, by virtue of being ‘a most just and wise king’, and his rank being higher in
the eyes of God, than that of the Mujtaddids, was entitled to choose any one of the interpretations, which
would be of benefit to the nation and in the interests of good order. Further, if Akbar issued a new order
‘inconformity with the Quran and calculated to benefit the nation’, all should be bound by it.
 The declaration, which was signed by the leading Ulamas, had been wrongly called a ‘Decree of Infallibility’.
Akbar claimed the right to choose only when there was a difference of opinion among those qualified to
interpret the Quran. At a time when there were bloody conflicts between the Shias, the Sunnis and the
Mahdawis indifferent parts of the country, Akbar wanted the widest toleration. There is little doubt that the
Mahzar had a salutary effect in stabilizing the religious situation in the empire.

Q 77. (B)
 The institution of Iqta had been in force in the early Islamic world, as a form of reward for services to the
state. In the Caliphate administration, it was used to pay civil and military officers. After the establishment
of the Sultanate, the Iqta system was introduced by the Sultans. To begin with, the army commanders and
nobles were given territories to administer and collect the revenue. The territories thus assigned were called
Iqtas and their holders as Iqtadar or Muqti.
 In essence, this was a system of payment to the officers and maintenance of army by them. Gradually,
rules and regulations were laid down to organize the whole system. Through the years it became the main
instrument of administrating the Sultanate.
 Further, the Sultans could get a large share of the surplus production from different parts of the vast
territories through this system. From the 14thcentury, we hear of Walis or Muqtis, who are commanders of
military and administrative tracts called Iqta. Their exact powers varied according to the circumstances. In
due course, the Muqti was given complete charge of the administration of the Iqta, which included the task
of maintaining an army. The Muqti was to help the Sultan with his army in case of need. He was expected to
maintain the army and meet his own expenses with the revenue collected.
 From the time of Balban, the Muqtiwas expected to send the balance (Fawazil) of the income to the Centre,
after meeting his and the army’s expenses. This means that the central revenue department had made an
assessment of the expected income of the Iqta, the cost of the maintenance of the army and the Muqti’s
own expenses. This process became even more strict during the time of Alauddhin Khalji.
 As the central control grew, the control over Muqti’s administration also increased. The Khwaja (probably
same as Sahib-i-Diwan) was appointed to keep a record of the income of the Iqtas. It was on the basis of this
record that the Sultan used to make his revenue demands. A Barid or intelligence officer was also appointed
to keep the Sultan informed. During the reign of Muhammad-bin-Thughlaq, a number of governors were
appointed on revenue sharing terms where they were to give a fixed sum to the state. During the time of
Feroze ShahTughlaq, the control of the state over Iqtas was diluted, when Iqtas became hereditary.

Q 78. (C)
22

 The most important department of the state, next to the Wazir’s, was the Diwan-i-Arz or the military
department. The head of this department was called the Ariz-i-Mamalik.
Page

Test: 17 (2020) | Test Code: A-20117 Contact us at www.iasprayojan@gmail.com / +91-7558644556


www.iasprayojan.com IAS PRAYOJAN NAGPUR
 The Ariz was not the commander in-chief of the army, since the Sultan himself commanded all the armed
forces. In those days, no king could have survived on the throne, if he entrusted the chief command of the
armed forces to someone else. The special responsibility of the Ariz’s department was to recruit, equip and
pay the army.
 There were two other important departments of the state: the Diwan-i-Risalat and the Diwan-i-Insha. The
former dealt with religious matters, pious foundations and stipends to the deserving scholars and the men
of piety. It was presided over by the Chief Sadr, who was generally a leading Qazi.
 The Diwan-i-Insha dealt with state correspondence. All the correspondence, formal or confidential,
between the ruler and the sovereigns of other states, and with his subordinate officials, was dealt with by
this department.
 The rulers posted intelligence agents, called Barids, in different parts of the empire to keep them informed
of what was going on.

Q 79. (B)
 Karikala was an important Chola ruler of the Sangam period, who was famous for his military achievements.
Vijayalaya, on the other hand, was the founder of the Imperial Chola Dynasty that began in the medieval
period (9thcentury). He is accredited with reestablishing the Chola rule and had his capital in Thanjavur.
 Pattinappalai portrays Karikala’s early life and his military conquests. In the Battle of Venni, he defeated
the mighty confederacy consisting of the Cheras, Pandyas and eleven minor chieftains. This event is
mentioned in many Sangam poems. Vahaipparandalai was another important battle fought by him, in which
nine enemy chieftains submitted before him. Karikala’s military achievements made him the overlord of the
whole Tamil country.
 Hereditary monarchy was the form of government during the Sangam period. The king took advice of his
minister, court-poet and the imperial court or avai. The Chera kingsassumed titles like
Vanavaramban,Vanavan, Kuttuvan, Irumporai and Villavar; the Chola kings like Senni, Valavan and Killi; and
the Pandyakings Thennavar and Minavar.

Q 80. (C)
 The Amara-Nayaka system was a major political innovation of the Vijayanagar Empire. It is likely that many
features of this system were derived from the Iqta system of the Delhi Sultanate.
 The Amara-Nayakas were military commanders, who were given territories to govern by the Raya. They
collected taxes and other dues from the peasants, crafts persons and traders in the area. They retained part
of the revenue for personal use and for maintaining a stipulated contingent of horses and elephants. These
contingents provided the Vijayanagar kings with an effective fighting force, with which they brought the
entire southern peninsula under their control.

Q 81. (D)
There were 4 important Buddhist Councils that were held for the propogation of the religion:

Council – 1st Buddhist Council


Place – Rajgriha
President – Mahakasyapa
Patron King – King Ajatshatru
Outcome – The teachings of Buddha weredivided into three Pitakas and the SuttaPittak and Vinay Pittak
were compiled.

Council – 2nd Buddhist Council


Place – Vaishali
23

President – Sabakami
Patron King – King Kalasoka
Page

Outcome – Discussed the Buddhistmonastic practices.


Test: 17 (2020) | Test Code: A-20117 Contact us at www.iasprayojan@gmail.com / +91-7558644556
www.iasprayojan.com IAS PRAYOJAN NAGPUR
Council – 3rd Buddhist Council
Place – Pataliputra
President – Moggliputta Tissa
Patron King – King Ashoka
Outcome – The third Pittak was composed

Council – 4th Buddhist Council


Place – Kundalrana, Kashmir
President – Vasumitra
Patron King – King Kanishka
Outcome – Buddhism was divided into two sects- Mahayana and Hinayana. Mahayana (Greater Vehicle)
worshipped various images of Buddha with the advent of the Christian era. The Hinayana sect of Buddhism
(Lesser Vehicle) worshipped the symbols of Gautam Buddha, like the lotus, bull, horse, wheel, Bodhi tree
etc.

Q 82. (B)
• Abul Fazl, while writing in the Ain-i-Akbari, speaks of fourteen silsilahs of the Sufis. These silsilahs were
divided into two types: Ba-shara and Beshara.
• Ba-shara was those orders that followed the Islamic Law (Sharia) and its directives, such as namaz and roza.
Chief amongst these were the Chishti, Suhrawardi, Firdawsi, Qadiri and Naqshbandi silsilahs.
• The Be-shara silsilahs were not bound by the Sharia. The Qalandars belonged to this group.
• Sheikh Fariduddin Ganj-i-Shakar of Ajodhan (Pattan in Pakistan) popularised the Chishti silsilah in modern
Haryana and Punjab. Heopened his door of love and generosity to all. Baba Farid, as he was called, was
respected by both Hindus and Muslims. His verses, written in Punjabi, are quoted in the Adi Granth.
• The term Kalam refers to the orthodox doctrines that prevailed in Islam. Kalam were challenged by the
rationalist arguments developed by the Ashari School.

Q 83. (D)
 Gopala founded the Pala empire in 750AD.
 The Rashtrakutas: 752-973
 Dantidurg (752-756) founded the kingdom. He fixed his capital atMalkhand.
 Mayurasharman fonded the Kadambas of Vanavasi.
 The dynasty of Gurjar Pratiharas wasfounded by Harichandra in 6th century AD.
 Guptas of Magadha was founded by Shri Gupta.

Q 84. (C)
 Statement 1 is incorrect: Virashaiva emerged in 12th century in Karnataka, led by a saint named
Basavanna.
 Statement 3 is incorrect: The ideas of Tamil bhaktas (especially Vaishnavas)were incorporated within
Sanskrit tradition which led into culmination of Bhagvata Purana.
 Statement 4 is incorrect: They ceremonially buried their dead but they questioned the theory of rebirth.
Supplementary notes:
Virashaiva Tradition
 The 12th century witnessed the emergence of a new movement in Karnataka, led by a Brahmana named
Basavanna (1106-68) who was a minister in the court of a Kalachuri ruler.
 His followers were known as Virashaivas(heroes of Shiva) or Lingayats (wearersof the linga). They worship
Shiva in his manifestation as a linga, and men usuallywear a small linga in a silver case on a loop strung over
the left shoulder.
 Lingayats believe that on death the
24

devotee will be united with Shiva and will not return to this world. Therefore they do not practice funerary
Page

rites such as cremation, prescribed in the Dharmashastras. Instead, they


Test: 17 (2020) | Test Code: A-20117 Contact us at www.iasprayojan@gmail.com / +91-7558644556
www.iasprayojan.com IAS PRAYOJAN NAGPUR
ceremonially bury their dead.
 The Lingayats challenged the idea of caste and the “pollution” attributed to certain groups by Brahmanas
thus winning them followers amongst those who were marginalized within the Brahmanical social order.
 They also questioned the theory of rebirth.
 The Lingayats also encouraged certain practices disapproved in the Dharmashastras, such as post-
puberty marriage and the remarriage of widows.
 Most of the understanding of the Virashaiva tradition is derived from vachanas (literally, sayings)
composed in Kannada by women and men who joined the movement.
 This period also witnessed another major development. Many ideas of Tamil bhaktas (especially the
Vaishnavas) were incorporated within the Sanskrit tradition, culminating in the composition of the one-
of the best known Puranas, the Bhagvata Purana.

Q 85. (D)
Important officials in the Mauryan Empire
Name Occupation
1. Sannidhata Director of Treasury
2. Panyaadhyaksha Director of trade
3. Pautavadhyaksha Director of weights and measure
4. Samsthadyaksha Director of market
5. Lavanadhyaksha Director of salt making
6. Sulkaandhyaksha Director of salt making
7. Sitaadhyaksha Director of land
8. Suradhyaksha Director of liquor manufacture

Q 86. (B)
 Statement 1 is incorrect: Business Class: The economic boycott received support from the Indian business
group because they had benefited from the nationalists’ emphasis on the use of Swadeshi. But a section of
the big business houses remained skeptical towards the movement. They seemed to be afraid labor unrest
in their factories.
 Statement 4 is incorrect: Students became active volunteers of the movement and thousands of them left
government schools and colleges and joined national schools and colleges.
Non-Cooperation Movement and Khilafat movement
The participation in the movement was from a wide range of the society but to a varying extent.
 Middle Class: People from the middle classes led the movement at the beginning but later they showed a
lot of reservations about Gandhi’s programme. The response to the call for resignation from the government
service, surrendering of titles, etc., was not taken seriously.
 Business Class: The economic boycott received support from the Indian business group because they had
benefited from the nationalists’ emphasis on the use of Swadeshi. But a section of the big business houses
remained skeptical towards the movement. They seemed to be afraid labor unrest in their factories.
 Peasants: Peasants’ participation was massive. In general, the peasants turned against the landlords and
the traders. The movement gave an opportunity to the toiling masses to express their real feelings against
the British as well as against their Indian masters and oppressors (landlords and traders).
 Women: Women gave up purdah and offered their ornaments for the Tilak Fund. They joined the
movement in large numbers and took active part in picketing of the shops selling foreign cloth and liquor.
 Hindu-Muslim Unity: The massive participation of Muslims and the maintenance of communal unity.
Gandhi and other leaders addressed the Muslim masses from mosques, and Gandhi was even allowed to
address meetings of Muslim women in which he was the only male who was not blind-folded.
 Students: Students became active volunteers of the movement and thousands of them left government
schools and colleges and joined national schools and colleges. The newly opened national institutions like
25

the Kashi Vidyapeeth, the Gujarat Vidyapeeth and the Jamila Milia Islamia and others accommodated many
Page

students.

Test: 17 (2020) | Test Code: A-20117 Contact us at www.iasprayojan@gmail.com / +91-7558644556


www.iasprayojan.com IAS PRAYOJAN NAGPUR
Q 87. (D)
Persian invasion
 The Indo-Iranian contact lasted for about200 years. Iranian influence may also be traced in the preamble
to Ashoka’s edicts as well as in certain terms used in them. For instance, for the Iranian term dipi, the
Ashokan scribe used the term lipi. Also, it appears that through the Iranians, the Greeks learnt about the
great wealth of India, which whetted their greed and led to Alexander’s invasion of India.
 The western borderland of India comprising the Punjab, Sindh and Afghanistan did not have any strong
political power during this period.
 Of the sixteen Mahajanapadas mentioned in the literature, only two, Kamboja and Gandhara, may be
placed in this outlying region. It appears to have been divided into a large number of independent
principalities which were frequently at war with one another, and thus an easy prey to foreign invaders.
 The powerful Achaemenian kings of Persia naturally cast their eyes towards this region, and perhaps Cyrus
(558- 530 B.C.) subjugated a number of principalities living to the south of the Hindukush mountains (parts
of India). It was in the reign of Darius (522-486 B.C.) that we have positive evidence of the extension of
Achaemenian rule in the northwestern part of India. Two inscriptions of this monarch mention “Hi(n)du” as
a part of his dominion.
 The exact connotation of this term is not known, but it certainly comprised some territory to the east of
the Sindhu, which Darius must have conquered about 518B.C. Herodotus, the Greek historian, tells us that
in 517 B.C. Darius sent a naval expedition to explore the valley of the Sindhu river
 The Iranian ruler Darius penetrated northwest India in 516 BC and annexed the Punjab (earlier Gandhara),
west of the Indus, and Sindh. This area was converted into the twentieth province or satrapy of Iran, which
had a total number of twentyeight satrapies.
 The Indian satrapy included Sindh, the north-west frontier, and the part of Punjab that lay to the west of
the Indus.It was the most fertile and populous part of the empire. It paid a tribute of 360 talents of gold,
which accounted for one third of the total revenue Iran received from its Asian provinces. The Indian
subjects were also enrolled in the Iranian army.

Q 88. (C)
Astadhyayi
 Panini wrote a descriptive linguistic treatise called “Astadhyayi” which covered the whole of Sanskrit
language’s grammar (including for the Vedic part) and also word formation rules in “sutra” format.

Q 89. (B)
 Statement 1 is incorrect: Krishnadeva Raya belonged to the Tuluva dynasty of the Vijayanagara Empire.
 Statement 3 is incorrect: Abur Razzaq and Conti visited during the reign of DevaRaya II.
Krishnadeva Raya
 He was the greatest king of the Vijaynagar empire. He was a great warrior and military commander. He
belonged to the Tuluva dynasty.
 His rule was characterized by expansion and consolidation. Under his rule, the land between the
Tungabhadra and Krishna rivers (the Raichur doab) was acquired(1512), the rulers of Orissa were
subdued(1514), and severe defeats were inflicted on the Sultan of Bijapur (1520). Although the kingdom
remained in a constant state of military preparedness, it flourished under conditions of unparalleled peace
and prosperity.
 He is credited with building some fine temples and adding impressive gopurams to many important South
Indian temples. He also founded a suburban township, after his mother, called Nagalapuram (near
Vijayanagara).
 He is also known as ‘Andhra Bhoja’, for his patronage to art and literature. He was a scholar and also known
as Abhinava Bhoja. He wrote the Telugu work ‘Amuktamalyada’ and a Sanskrit play ‘Jambavati Kalyana’.
 Foreigners who visited the Vijaynagara empire during his tenure were Barbosa and Paes while others like
26

Razzaq and Conti visited during the tenure of DevaRaya II.


Page

Test: 17 (2020) | Test Code: A-20117 Contact us at www.iasprayojan@gmail.com / +91-7558644556


www.iasprayojan.com IAS PRAYOJAN NAGPUR
Q 90. (D)
 Statement 2 is incorrect: Ashoka spoke of several features of Dhamma in his edicts. He did not clearly
define what Dhamma was.
Ashokan Edicts
 The great Emperor Ashoka, the third monarch of the Maurya dynasty converted to Buddhism after
witnessing the horrific effects of war in Kalinga. He became a champion and patron of Buddhism and strove
to spread Dhamma throughout his empire and beyond.
 He erected pillars and edicts all over the subcontinent and even in modern-day Afghanistan, Nepal,
Bangladesh, and Pakistan to spread the Buddha’s word. James Prinsep, a British antiquary and colonial
administrator was the first person to decipher Ashoka’s edicts. These inscriptions are the first tangible
evidence of Buddhism.
 More than religious discourses, they talk about the moral duties of the people, how to conduct life,
Ashoka’s desire to be a good and benevolent ruler, and about Ashoka’s work towards this end.
 Asoka used Dhamma as an ideology for ensuring coalescence in the empire. Dhamma became the central
theme of the edicts issued by Asoka.
 He categorically called himself an upasaka or a lay Buddhist devotee. The minor rock edict at Maski more
specifically labels him as a lay worshipper of the Buddha. The edicts leave little room for doubt about Asoka’s
personal leaning to Buddhism.
 Asoka used edicts to explain various features of the Dhamma such as respect towards elders, generosity
towards Brahmanas and those who renounced worldly life, treating slaves and servants kindly, and respect
for religions and traditions other than one ‘sown.
 Asoka only spoke of Dhamma’s various features rather than defining it.

Q 91. (A)
Non-Cooperation Movement
 The Non-Cooperation Movement launched by Mahatma Gandhi on August 1, 1920, was the first mass
movement organized nationwide during India’s struggle for freedom.
 In July 1921, the All-India Khilafat Committee passed a resolution declaring that no Muslim should serve
in the British Indian army.
 The Khilafat meetings in Malabar incited communal feelings among the Moplahs and it became a
movement directed against the British as well as the Hindu landlords of Malabar. From August 1921 till about
the end of the year, the rebels had under their control large parts of Malabar.
 The British government arrested Mahatma Gandhi on 10 March 1922and charged him with spreading
disaffection against the Government.
 In November 1921 huge demonstrations greeted the Prince of Wales, heir to the British throne, during
his tour of India. He had been asked by the Government to come to India to encourage loyalty among the
people and the princes. In Bombay, the Government tried to suppress the demonstration, killing 53 persons
and wounding about 400 more.

Q 92. (D)
Stagnation and Deterioration of Agriculture
 Overcrowding of agriculture, excessive land revenue demand, growth of landlordism, increasing
indebtedness, and the growing Impoverishment of the cultivators were few reasons which led to stagnation
and deterioration of agriculture.
 Indian agriculture began to stagnate and even deteriorate resulting in extremely low yields per acre.
 Overcrowding of agriculture and an increase in subinfeudation led to subdivision and fragmentation of
land into small holdings most of which could not maintain their cultivators.
 The extreme poverty of the overwhelming majority of peasants left them without any resources with which
to improve agriculture by using better cattle and seeds, more manure and fertilizers, and improved
27

techniques of production. Nordic the cultivator, rack-rented by both the Government and the landlord, have
any incentive to do so.
Page

Test: 17 (2020) | Test Code: A-20117 Contact us at www.iasprayojan@gmail.com / +91-7558644556


www.iasprayojan.com IAS PRAYOJAN NAGPUR
 After all, the land he cultivated was rarely his property and the bulk of the benefit which agricultural
improvements would bring was likely to be reaped by the horde of absentee landlords and money-lenders.
Subdivision and fragmentation of land also made it difficult to effect Improvements.

Q 93. (B)
 Option (b) is correct: In 1809 Rammohan Roy wrote in Persian his famous work Gift to Monotheists.
Raja Rammohan Roy
 Raja Ram Mohan Roy was an Indian socio educational reformer who was also known as Father of Modern
India’ and ‘Father of the Bengal Renaissance’.
 In 1809 Ram mohan Roy wrote in Persian his famous work Gift to Monotheists.
 Atmiya Sabha was started in 1815. It was a philosophical discussion circle in India where people used to
conduct debate and discussion sessions on philosophical topics such as Hindu Vedantism and also used to
promote free and collective thinking and social reform.
 Brahmo Samaj was established in 1828, whose purpose was to purify Hinduism and to preach theism or
the worship of one God. The society was to be based on the twin pillars of reason and the Vedas and
Upanishads.
 From 1818, he started a long crusade against the evils of Sati custom. He set out to rouse public opinion
on the question by citing the authority of the oldest sacred books that the Hindu religion right from its origin
is opposed to the practice of Sati.

Q 94. (B)
 Option (b) is correct:
List I List II
A. Sharia 3. Law governing Muslim community
B. Hadis 2. Traditions of the Prophet
C. Qiyas 4. Reasoning by analogy
D. Ijma 1. Consensus of the community
Islamic Traditions
 Ulama (plural of alim) were scholars of Islamic studies. As preservers of this tradition, they performed
various religious, juridical and teaching functions. They guided the Muslim rulers and ensured that they ruled
according to Sharia.
 Sharia is the law governing the Muslim community. It is based on the Quran and the hadis – the traditions
of the Prophet including a record of his remembered words and deeds.
 With the expansion of Islamic rule outside Arabia, in areas where customs and traditions were different,
qiyas (reasoning by analogy) and ijma (consensus of the community) were recognized as two other sources
of legislation.
 Thus the Sharia evolved from Quran, Hadis, qiyas and ijma.

Q 95. (B)
 Statement 1 and 4 are incorrect: Thereof the legislative council members resigned in protest to the act.
They were Mohommed Ali Jinnah, Madan Mohan Malaviya and Mazhar-ul-Huq.
Rowlatt Act of 1919
 British government enacted the Anarchical and Revolutionary Crimes Act of 1919,popularly known as the
Rowlatt Act. The act was against the basic principle of the rule of law.
 The act was hurriedly passed in the Imperial Legislative Council despite the united opposition of the Indian
members.
 It gave the government enormous powers to repress political activities and allowed the detention of
political prisoners without trial for two years.
28

 The Act would thus also enable the Government to suspend the right of habeas Corpus which had been
the foundation of civil liberties in Britain.
Page

Test: 17 (2020) | Test Code: A-20117 Contact us at www.iasprayojan@gmail.com / +91-7558644556


www.iasprayojan.com IAS PRAYOJAN NAGPUR
 Three of the legislative councilmembers resigned in protest to the act. They were Mohammed Ali Jinnah,
Madan Mohan Malaviya and Mazharul-Huq.

Q 96. (D)
Bhakti Movement in North India
 Historians have not found evidence of anything resembling the compositions of the Alvars and Nayanars
till the fourteenth century in North India.
 Various factors account for this:
 In north India this was the period when several Rajput states emerged. In most of these states Brahmanas
occupied positions of importance, performing a range of secular and ritual functions. There seems to have
been little or no attempt to challenge their position directly.
 At the same time other religious leaders like Naths, Siddhas and Jogis, whodid not function within the
orthodox Brahmanical framework, were gaining ground. Many of them came from artisanal groups, including
weavers, who were becoming increasingly important with the development of organized craft production.
[Demand for such production grew with the emergence of new urban centers, and long-distance trade with
Central Asia and West Asia].
 Many of these new religious leaders questioned the authority of the Vedas, and expressed themselves in
languages spoken by ordinary people.
 A new element in the situation was the coming of the Turks which culminated in the establishment of
the Delhi Sultanate (thirteenth century). This undermined the power of many of the Rajput states and the
Brahmanas who were associated with these kingdoms.

Q 97. (C)
 Option (c) is correct:
Philosophy Features
A. Samkhya 3. Rational and Scientific Theory
B. Yoga 1. Practice of control over pleasure and senses
C. Mimamsa 5. Performance of vedic sacrifices to attain salvation
D. Vaisheshika 2. Atom Theory
E. Vedanta 4. Considers Brahma as the only reality
6 schools of Philosophy
 By the beginning of the Christian era, six schools of philosophy developed.
 These were:
 Samkhya
 Samkhya, literally ‘count’, seems to have originated first.
 According to the early Samkhya philosophy, the presence of divine agency is not essential to the creation
of the world. The world owes its creation and evolution more to Nature or prakriti than to God. This was a
rational and scientific view.
 Around the 4th century AD, in
addition to prakriti, purusha or spirit was introduced as an element in the Samkhya system, and the creation
of the world was attributed to both. According to the new view, Nature and the spiritual element together
create the world. Thus, at the outset the Samkhya School of philosophy was materialistic, but later it tended
to become spiritualistic.
 A person can attain salvation through the acquisition of real knowledge, and his misery can be ended
forever.
 Yoga
 A person can attain salvation through meditation and physical application.
 Practice of control over pleasure, the senses, and bodily organs is central to this system.
 In order to obtain salvation, physical exercises in various postures called asanas are prescribed, and a
29

breathing exercise called pranayama is recommended.


 Nyaya
Page

Test: 17 (2020) | Test Code: A-20117 Contact us at www.iasprayojan@gmail.com / +91-7558644556


www.iasprayojan.com IAS PRAYOJAN NAGPUR
 Nyaya, or the school of analysis, was developed as a system of logic.
 Salvation can be attained through the acquisition of knowledge.
 The stress laid on the use of logic influenced Indian scholars who took to systematic thinking and reasoning.
 Vaisheshika
 The Vaisheshika School gives importance to the discussion of material elements or dravya. Earth, water,
fire, air, and ether (sky), whencombined, give rise to new objects.
 The Vaisheshika School propounded the atom theory believing that all material objects are made up of
atoms.
 The Vaisheshika thus marked the beginning of physics in India but the scientific view was diluted by a belief
in god and spiritualism, and this school put its faith in both heaven and salvation.
 Mimamsa
 Mimamsa literally means the art of reasoning and interpretation.
 However, reasoning was used to provide justifications for various Vedic rituals, and the attainment of
salvation was made dependent on their performance.
 According to the Mimamsa
School, the Vedas contain the eternal truth.
 In order to attain salvation, the Mimamsa School strongly recommended the performance of Vedic
sacrifices, which needed the services of priests and legitimized the social distance between the various
varnas.
 Vedanta
 Vedanta means the end of the Veda.
 Vedanta philosophy is traced to the earlier Upanishads. According to it, brahma is the reality and everything
else is unreal (Maya). The self (soul)or atma coincides with brahma. Therefore, if a person acquires the
knowledge of the self (atma), he acquires the knowledge of brahma, and thus attains salvation. Both
brahma and atma are eternal and indestructible.
 The theory of karma came to be linked to Vedanta philosophy.

Q 98. (D)
Montford reforms
 Indian nationalists were not satisfied with the advancement of constitutional rights under the Montford
reforms.
 They were no longer willing to let an alien government decide their fitness for self-government would they
be satisfied with the shadow of political power.
 The Indian National Congress met in a special session at Bombay in August 1918 under the presidentship
of Hasan Imam to consider the reform proposals. It condemned them as “disappointing and unsatisfactory”
and demanded effective self-government instead.

Q 99. (B)
Compositions of Alvar Saints
 Both Alvars and Nayanars claimed that their compositions were as important as the Vedas. For instance,
one of the major anthologies of compositions by the Alvars, the Nalayira Divyaprabandham, was frequently
described as the Tamil Veda.
 By 10th century, the compositions of 12 Alvars were compiled in an anthology known as the Nalayira
Divyaprabandham (or ‘Four Thousand Sacred Compositions’).
 The poems of Appar, Sambandar and Sundarar form the ‘Tevaram’, a collection that was compiled and
classified in the 10thcentury on the basis of the music of the songs.

Q 100. (A)
 Statement 2 is incorrect: Conditions in the South were more favorable to the English.
30

 Statement 3 is incorrect: In Eastern-India, the English Company had opened its first factories in Orissa in
Page

1633.

Test: 17 (2020) | Test Code: A-20117 Contact us at www.iasprayojan@gmail.com / +91-7558644556


FREE BOOKS, NOTES & VIDEOS FOR CIVILSERVICES

EBOOKS & UPSC PRELIMS USPC MAINS VIDEO FOR DAILY


MAGZINES MATERIALS MATERIALS CIVILSERVICES NEWSAPERS

SECUREIAS UPSC PRELIMS UPSC MAINS DELHI CIVILSERVICES


TESTSERIES TESTSERIES STUDENTS BOOKS

OPTIONAL SUBJECTS BOOKS, STATE PCS, SSC, BANKING


TEST SERIES, VIDEOS & NOTES BOOKS, TESTS VIDEOS & NOTES
1.GEOGRAPHY 1.UPPSC 2.SSC 3.MPSC
2.HISTORY 4.IBPS 5.RAS & RPSC
3.MATHEMATICS ENGINEERING BOOKS & MATERIAL
4. SOCIOLOGY 1. IES 2. GATE 3. IFoS
5.PUBLIC ADMINISTRATION 4. COMPUTER SCIENCE
6. POLITICAL SCIENCE 5. MECHINICAL ENGINEERING
7. ECONOMICS OTHER TELEGRAM CHANNELS
8 PHYSICS 1 GOVERNMENT JOBS
9 COMMERCE ACCOUNTANCY 2 LEARN YOGA & MEDITATION
10 ANTHROPOLOGY 3 LEARN ENGLISH
11 LAW 4 BEST DELAS & OFFERS
12 PHILOSOPHY 5 IAS HINDI BOOKS
13 CHARTERED ACCOUNTANTANCY 6 PDFs FOR ALL EXAMS
14 MEDICAL SCIENCE 7. WORLD DIGITAL LIBIRARY
1.CHENNAI STUDENTS 2.BANGLORE STUDENTS 3. CURRENT AFFAIRS
CONTACT FOR ADVERTISEMENT IN ABOVE CHANNLES
ADMIN1: ADMIN2:
www.iasprayojan.com IAS PRAYOJAN NAGPUR
The Growth of the East India Company’s Trade and Influence during 1600-1744
 The English East Company had very humble beginnings in India. Surat was the Centre of its trade till 1687;
throughout this period the English remained petitioner before the Mughal authorities.
 By 1623, they had established factories at Surat, Broach, Ahmedabad, Agra, and Masulipatam. From the
very beginning, the English trading company tried to combine trade and diplomacy with war and control of
the territory where their factories were situated.
 Conditions in the South were more favourable to the English as they did not have to face a strong Indian
Government there.
 The English opened its first factory in the South at Masulipatnam in 1611. But they soon shifted the centre
of their activity to Madras the lease of which was granted tothem by the local Raja in 1639. Here the English
built a small fort around their factory called ‘Fort St. George’.
 By the end of the 17th century the English Company was claiming full sovereignty over Madras.
 The Island of Bombay was acquired by East India Company from Portugal in 1668 and was immediately
fortified.
 In Eastern-India, the English Company had opened its first factory in Orissa in1633.
 It soon opened factories at Patna, Balasore, Dacca and other places in Bengal and Bihar.

31
Page

Test: 17 (2020) | Test Code: A-20117 Contact us at www.iasprayojan@gmail.com / +91-7558644556

You might also like